- PowerScore Staff
- Posts: 5972
- Joined: Mar 25, 2011
- Wed Apr 13, 2022 9:32 am
#94750
Complete Question Explanation
(The complete setup for this game can be found here: lsat/viewtopic.php?f=269&t=1940)
The correct answer choice is (D)
From the third rule, M does not stock aisle 1, and so answer choice (A) can be eliminated.
From the sequence created by the combination of the fifth and seventh rules, M cannot stock aisle 9, and so answer choice (B) can be eliminated.
The difference between answer choices (C), (D), and (E) is that (C) and (E) are incomplete lists of the aisles M could stock. As the question stem asks for a “complete list,” and answer choice (D) contains the complete list, it is the correct answer.
(The complete setup for this game can be found here: lsat/viewtopic.php?f=269&t=1940)
The correct answer choice is (D)
From the third rule, M does not stock aisle 1, and so answer choice (A) can be eliminated.
From the sequence created by the combination of the fifth and seventh rules, M cannot stock aisle 9, and so answer choice (B) can be eliminated.
The difference between answer choices (C), (D), and (E) is that (C) and (E) are incomplete lists of the aisles M could stock. As the question stem asks for a “complete list,” and answer choice (D) contains the complete list, it is the correct answer.
Dave Killoran
PowerScore Test Preparation
Follow me on X/Twitter at http://twitter.com/DaveKilloran
My LSAT Articles: http://blog.powerscore.com/lsat/author/dave-killoran
PowerScore Podcast: http://www.powerscore.com/lsat/podcast/
PowerScore Test Preparation
Follow me on X/Twitter at http://twitter.com/DaveKilloran
My LSAT Articles: http://blog.powerscore.com/lsat/author/dave-killoran
PowerScore Podcast: http://www.powerscore.com/lsat/podcast/